LSAT and Law School Admissions Forum

Get expert LSAT preparation and law school admissions advice from PowerScore Test Preparation.

 Administrator
PowerScore Staff
  • PowerScore Staff
  • Posts: 8917
  • Joined: Feb 02, 2011
|
#36691
Complete Question Explanation

Weaken. The correct answer choice is (D)

The drug company manager quoted here discusses a new product that is failing to sell. The manager
points out that a marketing campaign would not guarantee success, but would provide “one chance”
to rescue the failing product, and concludes that the company should try the campaign:
  • Premise: A marketing campaign would not be certain to work, but it would provide one
    chance to save a failing product.

    Conclusion: The company should try the new marketing campaign.
Note that the manager does not say that the advertising campaign is the “only” chance of saving the
campaign. Instead, the manager uses language indicating that the advertising campaign offers “a”
chance.
The stimulus is followed by a Weaken question, so the correct answer choice will hurt the author’s
argument, and somehow show that the suggested marketing campaign would be inadvisable—likely
referencing some sort of previously unmentioned detriment that would be associated with such a
campaign or perhaps the existence of better options for saving the drug.

Answer choice (A): Losses from any product are likely to be harmful to the company’s profits, but
this choice would strengthen the argument of the manager, who seems willing to try anything to
save this single failing product. This choice certainly does not weaken the manager’s argument, so it
cannot be the correct answer choice.

Answer choice (B): This answer choice provides little new information, because the drug company
manager has already pointed out that a marketing campaign’s success would not be guaranteed.

Answer choice (C): The manager said merely that the campaign was not guaranteed to be a success,
not that the campaign had no chance to succeed. Since this choice does not necessarily apply to the
manager’s suggested campaign, it does not weaken the argument presented in the stimulus.

Answer choice (D): This is the correct answer choice. If the marketing campaign suggested by
the manager would hurt the company’s overall position, then it would be much more difficult to
justify an expensive marketing campaign in order to have “one chance” at helping out a single failing
product.

Answer choice (E): The correct answer choice to this question should provide a reason to avoid the
suggested marketing campaign. This choice fails to do so, and should thus be ruled out.
 rachue
  • Posts: 140
  • Joined: Jun 22, 2011
|
#1551
Hi,

I'm having trouble understanding why D is correct here. The manager's argument is that the one way to save the newest product would be a new marketing campaign, and is the "one chance to save the product, so we should try it."

If that is so, how does D weaken that? D just says that this action could mean having to cutback in other campaigns to compensate, but that doesn't necessarily mean that the product wouldn't be saved, just that other products might be threatened. I don't see how it attacks the argument in the stimulus.

I hesitated between B and D and ultimately chose B because of this(although not confidently bc I didn't think B was that great of a choice either).

Thanks in advance.
 Steve Stein
PowerScore Staff
  • PowerScore Staff
  • Posts: 1153
  • Joined: Apr 11, 2011
|
#1566
The drug company manager quoted here discusses a new product that is failing to sell. The manager points out that a marketing campaign would not guarantee success, but would provide “one chance” to rescue the failing product, and concludes that the company should try the campaign:

Premise: A marketing campaign would not be certain to work, but it would provide one chance to save a failing product.

Conclusion: The company should try the new marketing campaign.

Note that the manager does not say that the advertising campaign is the “only” chance of saving the campaign. Instead, the manager uses language indicating that the advertising campaign offers “a” chance.

The stimulus is followed by a Weaken question, so the correct answer choice will hurt the author’s argument, and somehow show that the suggested marketing campaign would be inadvisable—likely referencing some sort of previously unmentioned detriment that would be associated with such a campaign or perhaps the existence of better options for saving the drug.

Answer choice B provides little new information, because the drug company manager already pointed out that a marketing campaign’s success would not be guaranteed.

Answer choice D is the correct answer choice. If the marketing campaign suggested by the manager would hurt the company’s overall position, then it would be much more difficult to justify an expensive marketing campaign in order to have “one chance” at helping out a single failing product.
 rachue
  • Posts: 140
  • Joined: Jun 22, 2011
|
#1583
That helps, thanks. I think I made a mistake here by not correctly identifying the premises and conclusion. If I had thought that over before, D would have been the obvious choice. Thanks for the help!
 eober
  • Posts: 107
  • Joined: Jul 24, 2014
|
#16662
Hi,

What should my prephrase be going to the answer choices? I thought "the answer choice will give a reason why a new marketing campaign for the newest product is not an option that could save the product". Is there a better prephrase? Also, did we eliminate answer choice B because it says "many new products fail" but at the same time stimulus does not state it is impossible for the product to fail if there is a marketing campaign it only states "it could be a way to save it"

Thanks for your time! :)
 Emily Haney-Caron
PowerScore Staff
  • PowerScore Staff
  • Posts: 577
  • Joined: Jan 12, 2012
|
#16705
Hi eober,

My pre-phrase here would be, "There is some reason why an opportunity to save the product by marketing is not necessarily a good thing." D provides that - that marketing campaign might give a chance to save the product, but would hurt the company overall, so it is not worth it.

And for B, we can eliminate it because there stimulus doesn't say the marketing campaign would guarantee success; it only says it would provide a chance.
 lanereuden
  • Posts: 147
  • Joined: May 30, 2019
|
#65598
Steve:

You say: Note that the manager does not say that the advertising campaign is the “only” chance of saving the campaign. Instead, the manager uses language indicating that the advertising campaign offers “a” chance.
If he had say “only” rather than “a”chance, would B then be correct? I mean, why is it important to note this?
Thanks
 Zach Foreman
PowerScore Staff
  • PowerScore Staff
  • Posts: 91
  • Joined: Apr 11, 2019
|
#66937
lanereuden,
So, if it were the only way, that would make the managers argument much stronger. It would be harder to weaken the argument. In this case, it would not change the answer. Even if it were the only way to save it, it still might not be worth the cost. B would still be wrong.
It is important to note, not because it changes the answer in this case, but because it is always important to note nuances of language on this test. It also leads one to a prephrase "there might be other ways to save the product, other than marketing". In this case, the prephrase is not terribly helpful, but again it is good to notice these things.
 180bound
  • Posts: 34
  • Joined: Jun 11, 2019
|
#76054
Hi, my question reiterates what some else had previously posted (I am still confused). The argument concludes that they "should try it" (the new marketing campaign) because it might save the product....The correct answer talks about the company's overall position being damaged by a new marketing campaign etc. My question is how does that weaken the argument? Could the marketing campaign not be successful while also necessitating cut backs on existing campaigns? I don't know too much about marketing so to me it would be superfluous to assume that a marketing campaign for 1 product couldn't be successful for 1 product while the company as a whole is "endangered". Could someone explain this further? To me getting this question correct depended on assumptions that one would HAVE to make about the nature of marketing.
 Adam Tyson
PowerScore Staff
  • PowerScore Staff
  • Posts: 5153
  • Joined: Apr 14, 2011
|
#76485
To weaken any argument, 180bound, your goal is to find the one answer that raises some doubts about the conclusion of the argument. You do not have to disprove it - that's "destroy" rather than "weaken." All you have to do is find a possible objection.

Here, if the conclusion is "we should do this," to weaken you want to find some objection to doing that, some potential problem that might result from doing that. Answer D weakens because it says "if we do that, it could cause a problem." If we accept answer D as true (which we must do, because the question stem told us to treat all the answers as true), it should make us pause and rethink our plans. Maybe this marketing thing is a bad idea, and so maybe we shouldn't do it? That pause, that moment of doubt, is what we need from a good weaken answer, and this one gives us plenty of reason to pause and reconsider our strategy.

By way of analogy, imagine you have a headache. A prescription pain medication might relieve your pain, but it isn't guaranteed. If you thought "I should try it, it might work," would you pause and reconsider if I told you it might also give you a heart attack? That's what D is doing - giving you a reason to pause and reconsider.

Get the most out of your LSAT Prep Plus subscription.

Analyze and track your performance with our Testing and Analytics Package.